Regra de L'Hôpital e Teorema do Confronto

Ver Todos os Exercícios

Teoria

A regra de L’Hôpital

Vamos relembrar essa regra para facilitar o cálculo do limite de uma sequência, ok?

Primeiro, vamos lembrar em quais casos podemos utilizar:

1) Indeterminações do tipo 0 0

2) Indeterminações do tipo ∞ ∞

Vamos ver a definição para relembrarmos melhor:

Vamos agora ver um exemplo:

Veja se a sequência abaixo converge ou diverge.

a n = ln ⁡ n n

Quando n vai para infinito, vemos que há uma indeterminação, assim, vamos aplicar a regra de L’Hôpital:

lim n → ∞ ⁡ ln ⁡ n n = lim n → ∞ ⁡ ( ln ⁡ n ) ' ( n ) ' = lim n → ∞ ⁡ 1 n 1 = 0

Logo essa sequência converge.

Muito mais simples, né? Então, todas as vezes que aparecer essas indeterminações, devemos pensar em L’Hôpital, porque em geral, a questão sairá mais facilmente.

O Teorema do Confronto

Vamos aqui enunciar o Teorema do confronto. Ele será útil no cálculo de muitos limites de sequências.

Sendo { a n } , { b n } e { c n } três sequências tais que a n < b n < c n , então, se a n e c n convergirem e se lim n → ∞ ⁡ a n = lim n → ∞ ⁡ c n = L , { b n } também converge e tem limite L .

Em resumo, se uma sequência está “entre” duas outras sequências que possuem limite L , então ela também possui o mesmo limite L . Esse teorema também é conhecido como “teorema do sanduíche”, talvez esse nome seja mais fácil de lembrar.

Vamos ver um exemplo para fixar melhor esse conceito:

Exemplo

Veja se a sequência abaixo converge ou diverge. No caso de convergir, encontre o valor:

a n =   cos ⁡ n n 2 + n

Vamos lá, o cosseno está sempre entre -1 e 1, não é mesmo?

- 1 ≤ cos ⁡ n ≤ 1

Então podemos escrever:

- 1 n 2 + n ≤ cos ⁡ n n 2 + n ≤ 1 n 2 + n ⇒ - 1 n 2 + n ≤ a n ≤ 1 n 2 + n

Ou seja, nossa sequência está entre outras duas. Vamos agora mostrar que essas duas sequências convergem para o mesmo valor. Aplicando o limite:

l i m n → ∞ 1 n 2 + n = 0   e l i m n → ∞ - 1 n 2 + n = 0

Assim:

0 ≤ l i m n → ∞ a n ≤ 0

Pelo Teorema do confronto, temos que a sequência converge para 0!

Sempre que aparecer cossenos e senos, pode ser interessante usar essa abordagem. Agora vamos praticar!

O Teorema do Confronto

Exercícios Resolvidos

Exercício Resolvido #1

Analise a convergência da sequência, no caso de convergir, calcule seu limite.

n e - n

Passo 1

Definindo

f x = x e - x = x e x

Temos:

f n = a n = n e - n

Passo 2

Como tanto o numerador como o denominador de f ( x ) tendem para infinito com x → ∞ , vamos calcular o limite aplicando a Regra de L’Hospital:

lim x → ∞ ⁡ x e x = lim x → ∞ ⁡ 1 e x = 0

Resposta

Portanto, lim n → ∞ ⁡ a n = 0 e a sequência converge.

Exercício Resolvido #2

Calcule, justificando, o limite da seguinte sequência:

x n = n n n

Passo 1

Vamos primeiro escrever isso de uma forma mais bacana:

x n = n 1 n 1 n = n 1 n 2

Humm, então o limite é:

lim n → ∞ ⁡ n 1 n 2

Passo 2

Então, aquela famosa propriedade de funções inversas, vamos usar isso pra exponencial e logaritmo neperiano:

f x = e ln ⁡ f x

Assim fica:

lim n → ∞ ⁡ e ln ⁡ n 1 n 2 = lim n → ∞ ⁡ e ln ⁡ n n 2

E a gente pode tirar o e de dentro do limite, fica assim, por conta de uma propriedade de limites

e l i m n → ∞ ln ⁡ n n 2

Passo 3

Então precisamos resolver o seguinte limite:

l i m n → ∞ ln ⁡ n n 2 = ?

Da pra ver a inderteminação de ∞ ∞ , logo, por L’Hospital:

l i m n → ∞ ln ⁡ n n 2 = lim n → ∞ ⁡ 1 / n 2 n =   l i m n → ∞ 1 2 n 2 = 0

Assim, voltando para o e lá do passo anterior:

lim n → ∞ ⁡ n 1 n 2 = e l i m n → ∞ ln ⁡ n n 2

lim n → ∞ ⁡ n 1 n 2 = e 0 = 1

Resposta

lim n → ∞ ⁡ n 1 n 2 = 1

Exercício Resolvido #3

Determine se a sequência a n n ∈ N converge, e em caso afirmativo, calcule o limite:

a n = 1 - 2 sen ⁡ 1 n 3 n

Passo 1

Beleza, pra uma sequência convergir, temos que:

lim n → ∞ ⁡ a n = L

Onde L é um número real, então é isso que precisamos verificar.

Passo 2

Então fica assim:

lim n → ∞ ⁡ 1 - 2 sen ⁡ 1 n 3 n = ?

Então aqui vai um paranauê, nós sabemos que logaritmo neperiano e exponencial são funções inversas uma da outra, então essa propriedade é válida:

f x = e ln ⁡ f x

“Pra que isso serve?” Vamos mostrar agora! Faça isso para a n :

lim n → ∞ ⁡ 1 - 2 sen ⁡ 1 n 3 n = lim n → ∞ ⁡ e ln ⁡ 1 - 2 sen ⁡ 1 n 3 n

“ E isso facilitou?!” Sim sim, olha essa propriedade do logaritmo:

ln ⁡ A n = n . ln ⁡ A

Vamos usar isso agora:

lim n → ∞ ⁡ e ln ⁡ 1 - 2 sen ⁡ 1 n 3 n = lim n → ∞ ⁡ e 3 n . ln ⁡ 1 - 2 sen ⁡ 1 n

Galerinha, a gente pode tirar o e de dentro do limite, fica assim, por conta de uma propriedade de limites

lim n → ∞ ⁡ e 3 n . ln ⁡ 1 - 2 sen ⁡ 1 n = e l i m n → ∞   ln ⁡ 1 - 2 sen ⁡ 1 n 1 3 n

Colocamos dessa forma porque agora virou uma indeterminação, 0 0 , dessa forma podemos aplicar o famoso L’Hospital!

Passo 3

Vamos só resolver o limite, mas não esqueça da exponencial! Deriva ae galera, lembrando que no numerador vão ter duas regras da cadeia. Uma para ln ⁡   com o s e n e outra para derivar o - 2 sen ⁡ 1 n :

l i m n → ∞   ln ⁡ 1 - 2 sen ⁡ 1 n 1 3 n = l i m n → ∞ 1 1 - 2 sen ⁡ 1 n . - 2 cos ⁡ 1 n - 1 n 2 - 1 3 n 2 = - 3 l i m n → ∞ 2 cos ⁡ 1 n 1 - 2 sen ⁡ 1 n

Então ficou fácil galera!

- 3 l i m n → ∞ 2 cos ⁡ 1 n 1 - 2 sen ⁡ 1 n = - 3   . 2.1 1 - 2.0 = - 6

Então fica, voltando para o e lá do passo anterior:

lim n → ∞ ⁡ 1 - 2 sen ⁡ 1 n 3 n = e l i m n → ∞   ln ⁡ 1 - 2 sen ⁡ 1 n 1 3 n

lim n → ∞ ⁡ 1 - 2 sen ⁡ 1 n 3 n = e - 6

Logo o limite existe! E é e - 6

Resposta

lim n → ∞ ⁡ 1 - 2 sen ⁡ 1 n 3 n = e - 6

Exercício Resolvido #4

Calcule o limite da sequência:

cos ⁡ n n    

Passo 1

Legal, temos uma sequência numérica de termo geral:

a n = cos ⁡ n n

Podemos observar que o cosseno (no numerador) é limitado, pois só varia entre - 1 e 1 .

Passo 2

Assim, vamos ter que:

- 1 ≤ cos ⁡ n ≤ 1

Então, dividindo todo mundo por n :

- 1 n ≤ cos ⁡ n n ≤ 1 n

Passo 3

Assim, aplicando limite nas expressões temos que:

lim n → ∞ ⁡ - 1 n = lim n → ∞ ⁡ 1 n = 0

Portanto, pelo teorema do Confronto:

L = 0

Resposta

L = 0 .

Exercício Resolvido #5

Determine se a seguinte sequência possui limite

a n = 4 n 2 + cos ⁡ n n 3 + 3

Passo 1

Vamos calcular o limite desse termo geral quando n → ∞

lim n → ∞ ⁡ a n

Como temos um cosseno ali, isso nos dá indícios que temos que usar o teorema do sanduiche.

- 1 ≤ cos ⁡ n ≤ 1

4 n 2 - 1 ≤ 4 n 2 + cos ⁡ n ≤ 4 n 2 + 1

4 n 2 - 1 n 3 + 3 ≤ 4 n 2 + cos ⁡ n n 3 + 3 ≤ 4 n 2 + 1 n 3 + 3

Vamos fazer os limites dos extremos

Passo 2

lim n → ∞ ⁡ 4 n 2 + 1 n 3 + 3 = lim n → ∞ ⁡ n 2 4 + 1 / n 2 n 3 1 + 3 / n 3 = lim n → ∞ ⁡ 4 + 1 / n 2 n 1 + 3 / n 3 = 4 ∞ = 0

A lógica é idêntica ao outro limite:

lim n → ∞ ⁡ 4 n 2 - 1 n 3 + 3 = lim n → ∞ ⁡ n 2 4 - 1 / n 2 n 3 1 + 3 / n 3 = lim n → ∞ ⁡ 4 - 1 / n 2 n 1 + 3 / n 3 = 4 ∞ = 0

Logo:

lim n → ∞ ⁡ 4 n 2 - 1 n 3 + 3 ≤ lim n → ∞ ⁡ 4 n 2 + cos ⁡ n n 3 + 3 ≤ lim n → ∞ ⁡ 4 n 2 + 1 n 3 + 3

0 ≤ lim n → ∞ ⁡ 4 n 2 + cos ⁡ n n 3 + 3 ≤ 0

Pelo teorema do sanduíche esse limite vai ser 0 e essa sequência converge.

Resposta

O limite dela é 0 .

Exercício Resolvido #6

Discuta a convergência da sequência a n = n ! / n n , onde n ! = 1 ⋅ 2 ⋅ 3 … n .

Passo 1

Em geral, quando se tem fatorial na expressão geral de uma sequência, é uma boa compará-la com outras sequências. Vamos tentar, então, aplicar o Teorema do Confronto. Para isso, precisamos reescrever a expressão do a n .

n ! n n = 1 n ⋅ 2 ⋅ 3 ⋅ 4 … n n ⋅ n ⋅ n … n ⏟ n - 1   termos

Já que n ! ≤ n n - 1 (pois n é o maior termo de n ! ), então:

2 ⋅ 3 ⋅ 4 … n n ⋅ n ⋅ n … n ≤ 1

Temos, assim:

n ! n n = 1 n 2 ⋅ 3 ⋅ 4 … n n ⋅ n ⋅ n … n ≤ 1 n

Certo, agora precisamos de um limite inferior. Já que n ! assim como n n devem ser positivos, podemos afirmar que:

0 ≤ n ! n n ≤ 1 n

Passo 2

O limite inferior já sabemos que é zero, agora precisamos calcular o superior.

lim n → ∞ ⁡ 1 n = 0

Como ambos os limites são zero, sabemos que lim n → ∞ ⁡ a n = 0 .

Resposta

lim n → ∞ ⁡ a n = 0 e a sequência converge.

Exercícios de Livros Relacionados

Demonstre que se underset(n - oo)(l i m) a_n = 0 e {b_n} for limitada, entãounderset(n - oo)(l i m) (a_n b_n) = 0
Teoria e exemplos - Teorema da sequência intercalada: Prove o “teorema da sequência intercalada” para as sequências: { a n } e { b n } convergem para L , então a sequência a 1 , b 1 , a 2 , b 2 , ⋅ ⋅
Teoria e exemplos: Prove o Teorema 2.
A sequência {n/(n + 1)} tem um menor limitante superior igual a 1. Mostre que se M é um número menor que 1, então os termos de {n/(n + 1)} finalmente excedem M. Sendo assim, se M  M sempre que nN. Com
Prove o Teorema 3.